Preptest 52 (September 2007) Logic Game #3, #16 Forum

Prepare for the LSAT or discuss it with others in this forum.
Post Reply
secretad

Bronze
Posts: 209
Joined: Tue Jan 04, 2011 11:26 pm

Preptest 52 (September 2007) Logic Game #3, #16

Post by secretad » Wed Apr 13, 2011 4:59 pm

This is an easy game, but this question is not easy to me.

I am not sure if I am misunderstanding the rules to sufficiently answer this question.

It asks which one cannot be the second seminar given on the second day of the sales training conference?

So, my first take, is I am very concerned about the wording of "second seminar." My set up is as follows:


_ _ _ (Short Seminars)
_ _ _ (Long Seminars)
1 2 3

If one were to make this set up as the base of the game, then one may have difficulty with #16 because of the wording of second seminars, where as on my diagram, short seminars are precluded from being second seminars. How can I prevent this from happening again in the future? There is nothing in the constraints or set up of this game to give an idea as to which should be the bottom or top of the advanced linear game.

User avatar
510Chicken

New
Posts: 89
Joined: Sat Jan 15, 2011 5:50 pm

Re: Preptest 52 (September 2007) Logic Game #3, #16

Post by 510Chicken » Wed Apr 13, 2011 11:21 pm

secretad wrote: So, my first take, is I am very concerned about the wording of "second seminar." My set up is as follows:


_ _ _ (Short Seminars)
_ _ _ (Long Seminars)
1 2 3

If one were to make this set up as the base of the game, then one may have difficulty with #16 because of the wording of second seminars, where as on my diagram, short seminars are precluded from being second seminars. How can I prevent this from happening again in the future? There is nothing in the constraints or set up of this game to give an idea as to which should be the bottom or top of the advanced linear game.
You cannot set up your diagram that way. It assumes that Long Seminars must come first on each day, which means you are imposing an additional constraint. But you know that. The solution is to simply not restrict which position short/long take.

_ _ _
_ _ _
1 2 3

That's fine. It's certainly enough to solve all the questions. The game doesn't tell you which must be on top/bottom because it's supposed to be open. The two levels only tell you which one comes first on any given day. They are not there to separate Long/Short specifically (and doing so gets you nothing).

_ _ | _ _ | _ _
1------2------3

This too is a legitimate diagram that doesn't misleadingly create two brackets of three. That said, I prefer it stacked as well. You just need to remember that a row of three does not necessarily correspond to one of the sets of three.

16.
You can eliminate (C) and (E) since they are interchangeable variables. (A) can be eliminated too, since "H" has no constraints and can always replace an earlier Long Seminar. That leaves (B) and (E). Check B first due to the nature of the constraint that relates the two answers.

P is 4th.
G/O must be 5/6 (Rule 1: One of each seminar type per day).
N must be 5/6 (Rule 3: N after P).
T must be 6 (Rule 2: T after G and O)
3 variables for 2 slots --> fail. So (B) is the correct response.

Post Reply

Return to “LSAT Prep and Discussion Forum”